Aristotle LSAT CR - Q 14

This topic has expert replies
Senior | Next Rank: 100 Posts
Posts: 75
Joined: Wed Feb 02, 2011 1:55 am
Thanked: 6 times
Followed by:3 members

Aristotle LSAT CR - Q 14

by Black Knight » Thu Feb 03, 2011 10:43 pm
Despite improvements in treatment for asthma, the death rate from this disease
has doubled during the past decade from its previous rate. Two possible
explanations for this increase have been offered. First, the recording of deaths due
to asthma has become more widespread and accurate in the past decade than it
had been previously. Second, there has been an increase in urban pollution.
However, since the rate of deaths due to asthma has increased dramatically even in
cities with long-standing, comprehensive medical records and with little or no urban
pollution, one must instead conclude that the cause of increased deaths is the use
of bronchial inhalers by asthma sufferers to relieve their symptoms.

Which one of the following is an assumption on which the argument depends?

(A) Urban pollution has not doubled in the past decade.

(B) Doctors and patients generally ignore the role of allergies in asthma.

(C) Bronchial inhalers are unsafe, even when used according to the recommended
instructions.

(D) The use of bronchial inhalers aggravates other diseases that frequently occur
among asthma sufferers and that often lead to fatal outcomes even when the
asthma itself does not.

(E) Increased urban pollution, improved recording of asthma deaths, and the use of
bronchial inhalers are the only possible explanations of the increased death rate
due to asthma.

OA after a few replies.

Legendary Member
Posts: 2330
Joined: Fri Jan 15, 2010 5:14 am
Thanked: 56 times
Followed by:26 members

by mundasingh123 » Fri Feb 04, 2011 1:30 am
Black Knight wrote:Despite improvements in treatment for asthma, the death rate from this disease
has doubled during the past decade from its previous rate. Two possible
explanations for this increase have been offered. First, the recording of deaths due
to asthma has become more widespread and accurate in the past decade than it
had been previously. Second, there has been an increase in urban pollution.
However, since the rate of deaths due to asthma has increased dramatically even in
cities with long-standing, comprehensive medical records and with little or no urban
pollution, one must instead conclude that the cause of increased deaths is the use
of bronchial inhalers by asthma sufferers to relieve their symptoms.
(C) Bronchial inhalers are unsafe, even when used according to the recommended
instructions.
(E) Increased urban pollution, improved recording of asthma deaths, and the use of
bronchial inhalers are the only possible explanations of the increased death rate
due to asthma.
OA after a few replies.
Its between C and E. But E deals more directly with the factors mentioned in the stimulus so it s E.The options have no bearing upon th econclusion when u negate them
I Seek Explanations Not Answers

User avatar
Legendary Member
Posts: 752
Joined: Sun Sep 12, 2010 2:47 am
Thanked: 20 times
Followed by:10 members
GMAT Score:700

by prachich1987 » Fri Feb 04, 2011 2:39 am
+1 for E
Thanks!
Prachi

User avatar
Master | Next Rank: 500 Posts
Posts: 208
Joined: Thu Aug 26, 2010 3:10 pm
Thanked: 20 times
Followed by:2 members

by hja379 » Fri Feb 04, 2011 5:16 am
Explanation:

Choice A: This is not what the argument assumes. Even if you negate this, the conclusion stands.
Choice B: Out of scope.
Choice C: 'unsafe' is too broad. Are they unsafe enough to cause asthma? Negating the statement does not weaken the conclusion.
Choice D: Irrelevant
Choice E: The conclusion states that since med records and pollution does not seem to be the cause, it has to be the bronchial inhalers. What if it is something else? What if it is the pollen in the air? If the 3 causes are not the possible reasons, then the conclusion is flawed. This must be the assumption and must be true.

Hope this helps.

User avatar
GMAT Instructor
Posts: 3225
Joined: Tue Jan 08, 2008 2:40 pm
Location: Toronto
Thanked: 1710 times
Followed by:614 members
GMAT Score:800

by Stuart@KaplanGMAT » Thu Mar 24, 2011 12:49 pm
Someone PM'd me about this question, so I'm resurrecting it, since it's a great example of a commonly occurring argument pattern.

Pattern recognition is one of the keys to speedy points on test day; after all, standardized tests like the GMAT are all about patterns.

In CR, one of the most frequently occurring argument patterns is causation. Any argument that can be boiled down to "x caused y", and there are lots of them on the GMAT, falls into this category.

This question is a perfect example. We can summarize the entire argument as:

A phenomenon occurred. I have ruled out 2 possible causes of the phenomenon. Therefore, this third cause must be responsible.

In every causal argument, the author is making the following assumption: there are no other possible causes for the event.

With that prediction in mind, we aggressively move through the choices. (E) clearly matches our prediction and we confidently select it, do a little happy dance and move on to the next question.

If this had been a weakening question, we would have looked for an answer that provided an alternative cause; if this had been a strengthening question, we would have looked for an answer that eliminated an alternative cause.

The moral of the story: learn the common patterns!
Image

Stuart Kovinsky | Kaplan GMAT Faculty | Toronto

Kaplan Exclusive: The Official Test Day Experience | Ready to Take a Free Practice Test? | Kaplan/Beat the GMAT Member Discount
BTG100 for $100 off a full course

Master | Next Rank: 500 Posts
Posts: 152
Joined: Wed Mar 12, 2008 4:36 pm
Thanked: 8 times
Followed by:2 members

by artistocrat » Sun Apr 03, 2011 5:08 pm
The author even spoonfeeds us with "first", "second", and third conclusions. Because "first" and "second" are out, third "MUST" be true. Therefore there are only three explanations. Choose E.

Master | Next Rank: 500 Posts
Posts: 139
Joined: Sat May 01, 2010 11:06 pm
Thanked: 4 times
GMAT Score:710

by badpoem » Tue Apr 12, 2011 9:41 am
E .

Legendary Member
Posts: 857
Joined: Wed Aug 25, 2010 1:36 am
Thanked: 56 times
Followed by:15 members

by AIM GMAT » Wed Apr 13, 2011 5:41 am
Nice question
Thanks & Regards,
AIM GMAT

Master | Next Rank: 500 Posts
Posts: 418
Joined: Sun Jul 04, 2010 12:48 pm
Thanked: 6 times
Followed by:3 members

by gmatdriller » Fri Oct 07, 2011 10:55 am
Hi "edirik,"
I doubt whether you have gone through the posts from both
"Hja379" and "Stuart". Their contributions are clear. If after
you have read their posts and you still have issues, please let
us know.

User avatar
Master | Next Rank: 500 Posts
Posts: 425
Joined: Wed Dec 08, 2010 9:00 am
Thanked: 56 times
Followed by:7 members
GMAT Score:690

by LalaB » Mon Oct 24, 2011 10:33 pm
yes, E is the best here.

User avatar
Master | Next Rank: 500 Posts
Posts: 416
Joined: Tue Aug 30, 2011 2:18 pm
Location: Delhi, India
Thanked: 13 times
Followed by:9 members

by vaibhavgupta » Tue Oct 25, 2011 5:35 am
Black Knight wrote:Despite improvements in treatment for asthma, the death rate from this disease
has doubled during the past decade from its previous rate. Two possible
explanations for this increase have been offered. First, the recording of deaths due
to asthma has become more widespread and accurate in the past decade than it
had been previously. Second, there has been an increase in urban pollution.
However, since the rate of deaths due to asthma has increased dramatically even in
cities with long-standing, comprehensive medical records and with little or no urban
pollution, one must instead conclude that the cause of increased deaths is the use
of bronchial inhalers by asthma sufferers to relieve their symptoms.

Which one of the following is an assumption on which the argument depends?

(A) Urban pollution has not doubled in the past decade.

(B) Doctors and patients generally ignore the role of allergies in asthma.

(C) Bronchial inhalers are unsafe, even when used according to the recommended
instructions.

(D) The use of bronchial inhalers aggravates other diseases that frequently occur
among asthma sufferers and that often lead to fatal outcomes even when the
asthma itself does not.

(E) Increased urban pollution, improved recording of asthma deaths, and the use of
bronchial inhalers are the only possible explanations of the increased death rate
due to asthma.

OA after a few replies.
E

Newbie | Next Rank: 10 Posts
Posts: 7
Joined: Mon Oct 03, 2011 5:54 am
Thanked: 1 times

by swetamurthy » Tue Oct 25, 2011 9:55 am
E.
good question
LSAT problems help in clear understanding
their practice has helped me a lot

Master | Next Rank: 500 Posts
Posts: 370
Joined: Sat Jun 11, 2011 8:50 pm
Location: Arlington, MA.
Thanked: 27 times
Followed by:2 members

by winniethepooh » Tue Oct 25, 2011 8:08 pm
Stuart... your explanations ROCK!!
Recognize patterns!
I wonder if I will be able to do that. :o

User avatar
Legendary Member
Posts: 626
Joined: Fri Dec 23, 2011 2:50 am
Location: Ahmedabad
Thanked: 31 times
Followed by:10 members

by ronnie1985 » Sat Apr 07, 2012 8:36 am
E is the answer
Follow your passion, Success as perceived by others shall follow you

Master | Next Rank: 500 Posts
Posts: 298
Joined: Tue Feb 16, 2010 1:09 am
Thanked: 2 times
Followed by:1 members

by Deepthi Subbu » Tue Apr 10, 2012 1:28 am
I was between C and E , but chose C just because E has this extreme word ' only ' compared to unsafe . I clearly understand Stuart's response , but am still not satisfied with the explanation . can someone help ?